Đến nội dung

quanrrom97 nội dung

Có 12 mục bởi quanrrom97 (Tìm giới hạn từ 25-04-2020)


Sắp theo                Sắp xếp  

#441000 $cos2x+sin2x=\sqrt{3}cos3x$

Đã gửi bởi quanrrom97 on 07-08-2013 - 12:11 trong Phương trình, Hệ phương trình Lượng giác

1) $cos2x+sin2x=\sqrt{3}cos3x$

2) $sin(4x+140^{\circ})=cot(2x-110^{\circ})$




#409344 $\frac{1}{2+\frac{1}{a}...

Đã gửi bởi quanrrom97 on 31-03-2013 - 08:38 trong Bất đẳng thức và cực trị

cm rằng với 3 số thực dương a,b,c sao cho abc=1 thi

    $\frac{1}{2+\frac{1}{a}}+\frac{1}{2+\frac{1}{b}}+\frac{1}{2+\frac{1}{c}}\leq 1$




#408677 tính A=tg20.tg80+tg80.tg140+tg140.tg20

Đã gửi bởi quanrrom97 on 28-03-2013 - 19:55 trong Công thức lượng giác, hàm số lượng giác

tính A=tg20.tg80+tg80.tg140+tg140.tg20




#403217 $(a\sqrt{b+c}+b\sqrt{a+c}+c\sqrt...

Đã gửi bởi quanrrom97 on 09-03-2013 - 13:50 trong Bất đẳng thức và cực trị

$(a\sqrt{b+c}+b\sqrt{a+c}+c\sqrt{a+b})^{2}\leq (a+b+c)(2ab+2bc+2ac)$



#401933 $(\sqrt{a}+\sqrt{b})^{8}\geq 64ab(a+b)^{2}$

Đã gửi bởi quanrrom97 on 04-03-2013 - 11:49 trong Bất đẳng thức và cực trị

cho các số dương a,b,x,y,z. Cmr:
1)
$(\sqrt{a}+\sqrt{b})^{8}\geq 64ab(a+b)^{2}$
2) $\sqrt[3]{a}+\sqrt[3]{b}\geq 2^{8}ab(a+b)$
3) $(\frac{x}{y}+\frac{y}{x})^{32}\geq 8^{9}(\frac{x^{2}}{y^{2}}+\frac{y^{2}}{x^{2}})^{4}(\frac{x^{4}}{y^{4}}+\frac{y^{4}}{x^{4}})$
4) $(\frac{x}{y}+\frac{y}{x})^{70}\geq 16^{16}(\frac{x^{3}}{y^{3}}+\frac{y^{3}}{x^{3}})^{5}(\frac{x^{5}}{y^{5}}+\frac{y^{5}}{x^{5}})$



#399975 $(2-a)(3-b)(4-c)(2a+3b+4c+3)\leq \frac{512}{3...

Đã gửi bởi quanrrom97 on 25-02-2013 - 20:16 trong Bất đẳng thức và cực trị

cmr:
1) với a,b thuộc [0:1] thì $(1-a)(1-b)(a+b)\leq \frac{8}{27}$
2) với a thuộc [-2;2], b thuộc [1/3:3], c thuộc [0:4] thì $(2-a)(3-b)(4-c)(2a+3b+4c+3)\leq \frac{512}{3}$



#399884 $(xyz+1)(\frac{1}{x}+\frac{1}...

Đã gửi bởi quanrrom97 on 25-02-2013 - 12:43 trong Bất đẳng thức và cực trị

bài 1: cho 3 số thực dương x,y,z. cmr:
$(xyz+1)(\frac{1}{x}+\frac{1}{y}+\frac{1}{z})+\frac{x}{z}+\frac{z}{y}+\frac{y}{x}\geq x+y+z+6$
bài 2: cho $x\in \mathbb{R^{+}}$ . cmr:
1) $x^{5}+x^{4}-3x^{3}+1\geq 0$
2) $x^{7}+x^{6}+3x^{5}-7x^{4}+2\geq 0$



#399283 $a^{2}b\leq \frac{500}{27}$

Đã gửi bởi quanrrom97 on 23-02-2013 - 10:37 trong Bất đẳng thức và cực trị

bài 1: cho a,b ko âm thoả đk $a+b\geq 5$ . cmr:
1) $a^{2}b\leq \frac{500}{27}$
2) $a^{2}b^{3}\leq 108$
bài 2: cho a,b,c ko âm thoả đk $a+b^{2}+c^{3}= 11$ . cmr:
1)$ab^{2}c^{3}\leq \frac{1331}{27}$
2)$abc\leq 6\sqrt[6]{108}$
bài 3: cmr:
1) với a,b thuộc [0:1] thì $(1-a)(1-b)(a+b)\leq \frac{8}{27}$
2) với a thuộc [-2;2], b thuộc [1/3:3], c thuộc [0:4] thì $(2-a)(3-b)(4-c)(2a+3b+4c+3)\leq \frac{512}{3}$
bài 4: cho 3 số thực dương x,y,z. cmr:
$(xyz+1)(\frac{1}{x}+\frac{1}{y}+\frac{1}{z})+\frac{x}{z}+\frac{z}{y}+\frac{y}{x}\geq x+y+z+6$
bài 5: cho $x\in \mathbb{R^{+}}$ . cmr:
1) $x^{5}+x^{4}-3x^{3}+1\geq 0$
2) $x^{7}+x^{6}+3x^{5}-7x^{4}+2\geq 0$
bài 6: cmr:
1) với a>1 thì $a+\frac{27}{2(a-1)(a+1)^{3}}\geq \frac{5}{2}$
2) với a,b,c>0 thoả a>b, a>c thì $2a+\frac{1}{(a-b)(a-c)(b+c)}\geq 4$
bài 7: cho 3 số ko âm a,b,c. cmr:
1) $(a^{2}+b^{2}+c^{2})(a+b)(b+c)(c+a)\leq 8\left ( \frac{a+b+c}{3} \right )^{9}$
2) $abc(a+b)^{2}(b+c)^{2}(c+a)^{2}\leq 64\left ( \frac{a+b+c}{3} \right )^{9}$
--
MOD:Tiêu đề của bạn đã đặt sai.Bạn tham khảo cách đặt tiêu đề tại đây



#399282 $abc(a+b)^{2}(b+c)^{2}(c+a)^{2}\leq 64\left ( \frac{a+b+c...

Đã gửi bởi quanrrom97 on 23-02-2013 - 10:34 trong Bất đẳng thức và cực trị

bài 1: cho a,b ko âm thoả đk $a+b\geq 5$ . cmr:
1) $a^{2}b\leq \frac{500}{27}$
2) $a^{2}b^{3}\leq 108$
bài 2: cho a,b,c ko âm thoả đk $a+b^{2}+c^{3}= 11$ . cmr:
1)$ab^{2}c^{3}\leq \frac{1331}{27}$
2)$abc\leq 6\sqrt[6]{108}$
bài 3: cmr:
1) với a,b thuộc [0:1] thì $(1-a)(1-b)(a+b)\leq \frac{8}{27}$
2) với a thuộc [-2;2], b thuộc [1/3:3], c thuộc [0:4] thì $(2-a)(3-b)(4-c)(2a+3b+4c+3)\leq \frac{512}{3}$
bài 4: cho 3 số thực dương x,y,z. cmr:
$(xyz+1)(\frac{1}{x}+\frac{1}{y}+\frac{1}{z})+\frac{x}{z}+\frac{z}{y}+\frac{y}{x}\geq x+y+z+6$
bài 5: cho $x\in \mathbb{R^{+}}$ . cmr:
1) $x^{5}+x^{4}-3x^{3}+1\geq 0$
2) $x^{7}+x^{6}+3x^{5}-7x^{4}+2\geq 0$
bài 6: cmr:
1) với a>1 thì $a+\frac{27}{2(a-1)(a+1)^{3}}\geq \frac{5}{2}$
2) với a,b,c>0 thoả a>b, a>c thì $2a+\frac{1}{(a-b)(a-c)(b+c)}\geq 4$
bài 7: cho 3 số ko âm a,b,c. cmr:
1) $(a^{2}+b^{2}+c^{2})(a+b)(b+c)(c+a)\leq 8\left ( \frac{a+b+c}{3} \right )^{9}$
2) $abc(a+b)^{2}(b+c)^{2}(c+a)^{2}\leq 64\left ( \frac{a+b+c}{3} \right )^{9}$
_____________________________
@Joker: Chú ý tiêu đề bạn nhé



#392538 $\frac{a}{b+2c+3d}+\frac{b}...

Đã gửi bởi quanrrom97 on 02-02-2013 - 19:00 trong Bất đẳng thức và cực trị

  • $\frac{a^{8}+b^{8}+c^{8}}{a^{3}b^{3}c^{3}}\geq \frac{1}{a}+\frac{1}{b}+\frac{1}{c}$ (a,b,c>0)
  • $\sum \frac{a}{2a+b+c}\leq \frac{3}{4}$ (a,b,c>0)
  • $\frac{9}{a+b+c}\leq \sum \frac{4}{2a+b+c}\leq \frac{1}{a}+\frac{1}{b}+\frac{1}{c}$ (a,b,c>0)
  • $\sqrt{x^{2}+x+1}+\sqrt{x^{2}-x+1}\geq 2$
  • $\sum \sqrt{\frac{a+b}{c}}\geq \sum 2\sqrt {\frac{c}{a+b}}$ (a,b,c>0)
  • $\frac{a}{b+2c}+\frac{b}{c+2a}+\frac{c}{a+2b}\geq 1$ (a,b,c>0)
  • $\frac{a}{b+2c+3d}+\frac{b}{c+2d+3a}+\frac{c}{d+2a+3b}+\frac{d}{a+2b+3c}\geq \frac{2}{3}$ (a,b,c,d>0)
  • CM nếu $\left | b \right |<\frac{\left | a \right |}{2}$ thì $\frac{1}{\left | a-b \right |} < \frac{2}{\left | a \right |}$
  • CM nếu $\left | a \right |\leq 2$ thì $x^{2}+axy+y^{2}\geq 0$ ( với mọi x,y)



#392535 $\frac{a}{b+2c+3d}+\frac{b}...

Đã gửi bởi quanrrom97 on 02-02-2013 - 18:53 trong Bất đẳng thức và cực trị

  • $\frac{a^{8}+b^{8}+c^{8}}{a^{3}b^{3}c^{3}}\geq \frac{1}{a}+\frac{1}{b}+\frac{1}{c}$ (a,b,c>0)
  • $\sum \frac{a}{2a+b+c}\leq \frac{3}{4}$ (a,b,c>0)
  • $\frac{9}{a+b+c}\leq \sum \frac{4}{2a+b+c}\leq \frac{1}{a}+\frac{1}{b}+\frac{1}{c}$ (a,b,c>0)
  • $\sqrt{x^{2}+x+1}+\sqrt{x^{2}-x+1}\geq 2$
  • $\sum \sqrt{\frac{a+b}{c}}\geq \sum 2\sqrt {\frac{c}{a+b}}$ (a,b,c>0)
  • $\frac{a}{b+2c}+\frac{b}{c+2a}+\frac{c}{a+2b}\geq 1$ (a,b,c>0)
  • $\frac{a}{b+2c+3d}+\frac{b}{c+2d+3a}+\frac{c}{d+2a+3b}+\frac{d}{a+2b+3c}\geq \frac{2}{3}$ (a,b,c,d>0)
  • CM nếu $\left | b \right |<\frac{\left | a \right |}{2}$ thì $\frac{1}{\left | a-b \right |} < \frac{2}{\left | a \right |}$
  • CM nếu $\left | a \right |\leq 2$ thì $x^{2}+axy+y^{2}\geq 0$ ( với mọi x,y)



#390645 $\frac{25a}{b+c}+\frac{16b}...

Đã gửi bởi quanrrom97 on 27-01-2013 - 10:14 trong Bất đẳng thức và cực trị

Chứng minh các BDT sau với a,b,c>0
  • $\left ( \frac{a+b}{a-b} \right )^{2}+\left ( \frac{b+c}{b-c} \right )^{2}+\left ( \frac{c+a}{c-a} \right )^{2}\geq 2$
  • $\frac{bc}{a^{2}b+a^{2}c}+\frac{ac}{b^{2}a+b^{2}c}+\frac{ab}{c^{2}a+c^{2}b}\geq \frac{1}{2a}+\frac{1}{2b}+\frac{1}{2c}$
  • $\frac{a^{5}}{a^{2}+ab+b^{2}}+\frac{b^{5}}{b^{2}+bc+c^{2}}+\frac{c^{5}}{a^{2}+ac+c^{2}}\leq \frac{a^{3}+b^{3}+c^{3}}{3}$
  • $\frac{25a}{b+c}+\frac{16b}{a+c}+\frac{c}{a+b}> 8$
Mod. Chú ý tiêu đề nhé